LSAT and Law School Admissions Forum

Get expert LSAT preparation and law school admissions advice from PowerScore Test Preparation.

 afeinstein
  • Posts: 1
  • Joined: Jun 26, 2014
|
#15112
I am having difficulty grasping how the answers should be determined for 5.1. Please assist.
 Lucas Moreau
PowerScore Staff
  • PowerScore Staff
  • Posts: 216
  • Joined: Dec 13, 2012
|
#15124
Hello, afeinstein,

The trick to this one is forming blocks. Look at how the first and second rules play out. They can be diagrammed like this:

FG or GF
HJ or JH

So that forms two blocks, the order of which we are not sure. Then if S must be fourth, we can build a diagram that looks like this:

_ _ _ S _ _ _

Looking at the last two rules means we cannot have GF, for G must be after F; and also we cannot have JH, for H must be before J. So we have an FG block, an HJ block, and S in the middle.

One of those blocks must come before S - that is, filling slots 1 and 2, or 2 and 3 - and one of those blocks must come after S - that is, filling slots 5 and 6, or slots 6 and 7.

This is why E is the correct answer choice. If Q and R are played consecutively, then there is no room for two other two-song blocks, because of S's placement. Therefore, Q and R cannot be played consecutively.

None of the other answer choices work, because it is the fact of there being a QR or RQ block that makes E unable to be true. There cannot be three two-song blocks with S splitting the remaining gaps into three-gap chunks.

Hope that helps,
Lucas Moreau

Get the most out of your LSAT Prep Plus subscription.

Analyze and track your performance with our Testing and Analytics Package.